Home
Current Affairs January 2024

What is the correct answer?

4

Topic: Tension headache.
Subject: Medicine
A woman presents with chief complaint of tightening, band-like headaches that usually occur upon rising in the morning. What is the most likely diagnosis?

A. Migraine headache

B. Premenstrual headache

C. Tension headache

D. Cluster headache

Correct Answer :

C. Tension headache


This is a classic presentation of a tension headache.

Common treatments for sinus, cluster, tension and migraine headaches are antibiotics, oxygen, NSAIDS and sumatriptan respectively.

Related Questions

What is the correct answer?

4

Topic: Seizure
Subject: Medicine
A 52-year-old male with a history of seizure attacks suffered from loss of consciousness and tonic-clonic muscular contractions. His tongue fell back into his throat and he choked.
What is the medication of choice for the treatment of this patient's condition?

A. Phenytoin

B. Valproic acid

C. Tiagabine

D. Phenobarbital

What is the correct answer?

4

Topic: Neurologic pain
Subject: Medicine
A 21-year-old man presented to the emergency hospital with severe abdominal pain. The pain started four hour ago. His medical history reveals Acute Intermittent Porphyria (AIP) and he used to have frequent attacks of abdominal pain and believes that this attack is no different. On examination, the patient is in acute pain and slightly feverish but not pale, jaundiced or cyanosed. However, his blood pressure is found to be high and he has tachycardia.
You should consider a diagnosis other than an AIP attack if the pain is associated with:

A. Constipation

B. Tenderness

C. Abdominal distension

D. Decreased bowel sounds

What is the correct answer?

4

Topic: Horner's Syndrome
Subject: Medicine
A 70-year-old man presented with ptosis, myosis and anhydrosis on the left side. Which one of the following is the most likely cause of this condition?

A. Tumour induced exophthalmos

B. Fourth cranial nerve palsy

C. Apical pulmonary carcinoma

D. Enlarged thyroid gland

What is the correct answer?

4

Topic: Cauda Equina Syndrome
Subject: Medicine
A 65-year-old male presents with a 1-month history of problems passing urine. He says that his bladder will feel full when he needs to urinate, but the urine stream is weak and his bladder does not feel as if it has emptied completely. The symptoms have become increasingly severe over the past week. Other symptoms include upper respiratory congestion for 3 days which he has treated with an over-thecounter decongestant with some relief, constipation with no passage of stool in the past 9 days, and increasing low back pain incompletely relieved with ibuprofen, with associated weakness in both legs. Examination shows a healthy-appearing male who is moderately overweight. He is afebrile and vital signs are normal. There is no abdominal tenderness and no masses are detected. A rectal examination reveals a large amount of hard stool in the rectum; a markedly enlarged (4+), boggy, tender prostate gland; laxity of the anal sphincter; and numbness in the perianal area. Urinalysis shows trace protein and 10-20 WBCs/hpf. Ultrasonography shows a post-void residual volume of 250 mL (normal for age <100).
Which one of the following must be done urgently in this patient?

A. Foley catheterization

B. Hospitalization for intravenous antibiotics

C. Digital disimpaction of the rectum, and Fleet enemas until clear

D. MRI of the lumbosacral spine

What is the correct answer?

4

Topic: Temporal Arteritis
Subject: Medicine
A 70-year-old woman returns to the office because of aching and weakness in her arms to the point where she cannot lift her arm to brush her hair. Physical examination shows no muscle tenderness or other evidence of joint disease in both arms. The aching improves when she takes the prescribed nonsteroidal anti-inflammatory drug (NSAID). She also describes tenderness over the right temporal area of her scalp. Physical examination of the scalp shows no lesions.
Which of the following is the most appropriate next step?

A. Increase the dose of the NSAID

B. Order determination of erythrocyte sedimentation rate

C. Order determination of serum rheumatoid factor

D. Order x-ray films of the cervical spine

What is the correct answer?

4

Topic: Glasgow Coma Scale
Subject: Medicine
A 27 year old man is brought into the ER after a bicycling accident. A car door suddenly opened in front of him, of which he smashed into and was thrown 15 feet. On examination, he is drowsy and confused. He opens his eyes when his name is called. When he speaks, you understand his words but the sentences do not make sense. He moves all four limbs but does not respond to any commands. He is able to pull both hands away when pinched and squirms when his sternum is rubbed, making no effort to stop you.
What is his Glasgow Coma Scale score?

A. 11

B. 10

C. 9

D. 8

What is the correct answer?

4

Topic: Temporal Arteritis
Subject: Medicine
A 75-year-old female presents with a 2-month history of bilateral headache, diffuse myalgias, and diplopia. On examination she has substantially diminished vision in her left eye, but no other neurologic findings. A moderately tender, cordlike structure is palpable just anterior to her ear and extending up to her lateral scalp. Blood tests show a markedly elevated erythrocyte sedimentation rate.
Which one of the following would be most appropriate at this point?

A. Clopidogrel (Plavix)

B. High-dose corticosteroids

C. NSAIDs

D. Dipyridamole/aspirin (Aggrenox)

What is the correct answer?

4

Topic: Horner's Syndrome
Subject: Medicine
A 66-year-old diabetic man presents with constriction of the pupil, drooping of the upper lid, and anhidrosis on the left. Which one of the following nerves is most likely involved?

A. Oculomotor

B. Sympathetic

C. Trochlear

D. Trigeminal

What is the correct answer?

4

Topic: Brown-Sequard Syndrome
Subject: Medicine
Which of the following would not be expected in a right-sided Brown-S�quard syndrome?

A. Right-sided hemi-paresis

B. Right-sided loss of proprioception

C. Left-sided decreased sensitivity to pinprick

D. Left-sided decreased vibration sense

What is the correct answer?

4

Topic: Seizure
Subject: Medicine
Of the following, which is the most frequent cause of seizures in the elderly?

A. Alcohol withdrawal

B. Stroke

C. Head trauma

D. Hypoglycemia

What is the correct answer?

4

Topic: Trigeminal Neuralgia
Subject: Medicine
A 55-year-old man complains of extremely severe, sharp, shooting pain in his face. He describes the episodes as being like a bolt of electricity that are brought about by touching a specific area, last about 60 seconds, and occur many times during the day. Neurologic examination is completely normal, but it is noted that part of his face is unshaven because he fears to touch that area. Gadoliniumenhanced MRI shows no abnormalities of the trigeminal nerve.
Which of the following is the most appropriate initial treatment?

A. Anticonvulsants

B. Aspirin

C. Nonsteroidal anti-inflammatory drugs

D. Vasoconstrictors

What is the correct answer?

4

Topic: Analgesics
Subject: Medicine
A 74-year-old black female has moderately severe pain due to osteoarthritis. However, she is also on medication for a seizure disorder. When choosing medications to manage her chronic pain, which one of the following should be used with caution because of her history of seizures?

A. Salsalate (Disalcid)

B. Celecoxib (Celebrex)

C. Hydrocodone (Lortab)

D. Tramadol (Ultram)

What is the correct answer?

4

Topic: Postherpetic Neuralgia
Subject: Medicine
A 78-year-old male comes to your office with a 3- day history of pain in the right side of his chest. The pain is described as burning and intense. Two days ago he noted a rash at that site. Examination reveals groups of vesicles on an erythematous base in a T-5 dermatome distribution on the right.
Which one of the following would be the most appropriate treatment to minimize the chance of post-herpetic neuralgia?

A. Famiciclovir (Famvir)

B. Prednisone

C. Capsaicin (Zostrix)

D. Carbamazepine (Tegretol)

What is the correct answer?

4

Topic: Korsakoff's Psychosis
Subject: Medicine
In a patient with recent onset of alcohol amnestic disorder, the mental status test most likely to be done poorly is memory of:

A. Political figures of 10 years ago

B. Digits (forward)

C. His own name

D. Three items for 5 minutes

What is the correct answer?

4

Topic: Amnesia
Subject: Medicine
Which of the following medications may cause amnesia as a side effect?

A. Hydroxyzine

B. Fluoxetine

C. Triazolam

D. Clonidine

What is the correct answer?

4

Topic: Bell's Palsy
Subject: Medicine
A 53-year-old male accountant comes to your office with progressive facial weakness on the left side that began yesterday. He also reports pain behind the left ear and decreased lacrimation from the left eye. He has been in good health and had his yearly physical examination 1 week ago, which was normal. His lipid levels, chemistry profile, and CBC were all normal. He has not been involved in any outdoor activities, nor does he engage in any high-risk sexual behavior. On examination, there is flattening of the left nasolabial fold and decreased ability to close the left eye. The mouth appears to be drawn to the right. The remainder of his general examination and neurologic examination are normal.
Which one of the following would be the most appropriate management at this time?

A. Carotid ultrasonography

B. High-resolution CT

C. MRI with gadolinium enhancement

D. Prednisone and valacyclovir (Valtrex)

What is the correct answer?

4

Topic: Guillain-Barr� syndrome
Subject: Medicine
A 43-year-old man presents 2 weeks after you see him for infectious diarrhea caused by C. jejuni. He has now developed bilateral proximal lower limb weakness and bilateral distal parasthesia and decreased ankle tendon reflex.
What is the most likely diagnosis?

A. Guillain-Barr� syndrome

B. Multiple Sclerosis

C. Myasthenia Gravis

D. Systemic Lupus Erythematosus

What is the correct answer?

4

Topic: Guillain-Barr� syndrome
Subject: Medicine
Guillain-Barr� syndrome is not associated with which of the following?

A. Proximal muscle weakness

B. Areflexia

C. Infection with C. jejuni

D. Treatment with corticosteroids

What is the correct answer?

4

Topic: Essential Tremor
Subject: Medicine
A 75 year old white male complains of a tremor which has been progressive over the past 2 years. The tremor interferes with writing, pouring liquids, and eating soup. He has no other medical problems. He abstains from alcohol and tobacco products. Physical examination is remarkable for an action tremor of the upper extremities and a head tremor. No rigidity or gait disorder is noted. Of the following agents, which one is most appropriate as initial drug therapy for this problem?

A. Alprazolam (Xanax)

B. Clonazepam (Klonopin)

C. Carbamazepine (Tegretol)

D. Propranolol (Inderal)

What is the correct answer?

4

Topic: Hepatic Encephalopathy
Subject: Medicine
A 50-year-old man with a history of hemochromatosis presents to the emergency room vomiting up bright red blood. He had his most recent phlebotomy yesterday. His blood pressure is 110/85 mm Hg, his pulse 115/min; his face is flushed, and he is diaphoretic. During the physical examination splenomegaly and a venous pattern on his chest and abdomen are noted. He seems somewhat drowsy and confused but has no focal neurologic signs.
What is the probable source of this patient's confusion?

A. Severe anemia

B. Hepatic encephalopathy

C. Subarachnoid hemorrhage

D. Vitamin B12 deficiency

What is the correct answer?

4

Topic: Osmotic Demyelination
Subject: Medicine
Osmotic demyelination can result when which one of the following is corrected too rapidly?

A. Hypocalcemia

B. Hypoglycemia

C. Hypomagnesemia

D. Hyponatremia

What is the correct answer?

4

Topic: Subarachnoid Hemorrhage
Subject: Medicine
A 36-year-old female presents with the sudden onset of severe headache, nausea, and photophobia. Her level of consciousness is progressively diminishing. Which one of the following would be the most appropriate next step?

A. Head CT without contrast

B. Head CT with contrast

C. Head MRI

D. Lumbar puncture

What is the correct answer?

4

Topic: Mini Mental Status Exam (or MMSE)
Subject: Medicine
In healthy adults, performance on the Folstein Mini-Mental State Examination is affected by which one of the following?

A. Educational attainment

B. Socioeconomic status

C. Gender

D. Race

What is the correct answer?

4

Topic: Fragile X Syndrome
Subject: Medicine
You are asked to see a mentally challenged 45-yearold male from a nearby group home who has groin pain. On examination you notice that he has large ears, a prominent jaw, and large symmetric testicles.
These findings are consistent with:

A. A variant form of Down syndrome

B. Aspergers syndrome

C. Klinefelters syndrome

D. Fragile X syndrome

What is the correct answer?

4

Topic: Seizure
Subject: Medicine
A 75-year-old male is brought to your office 1 month after a stroke that involved the left anterior cerebral artery, manifested by leg weakness, initial incontinence, and slowness in mentation. He experienced seizure activity on the second day after his stroke, but this was controlled by phenytoin (Dilantin). He has improved significantly and is now ambulatory. His family states that he now has episodic confusion, sleepiness, and clumsiness, which is preceded by paresthesias and dizziness, although no tonicclonic activity has been noted. He remains very drowsy for several hours after these episodes. He was wearing a cardiac monitor during one episode, but it showed nothing remarkable. His phenytoin level is therapeutic, and a CBC, metabolic profile, and magnesium level are all normal.
Which one of the following would be the most appropriate next step?

A. Discontinue the phenytoin

B. Add phenobarbital to the phenytoin

C. Begin bupropion (Wellbutrin)

D. Begin lamotrigine (Lamictal)

What is the correct answer?

4

Topic: DM neuropathy
Subject: Medicine
In which of the following diseases would you see a gloves and stocking neuropathy?

A. Diabetes mellitus

B. Lupus

C. Multiple sclerosis

D. ALS

What is the correct answer?

4

Topic: Intracranial Hemorrhage
Subject: Medicine
The most significant risk factor for spontaneous intracerebral hemorrhage is:

A. Atrial fibrillation

B. Cigarette smoking

C. Excessive alcohol use

D. Hypertension

What is the correct answer?

4

Topic: Adverse Drug Effect
Subject: Medicine
During the morning rounds, a second year resident presents a patient as a 58-year-old female who suffers from rigidity and tremors. She has been diagnosed with a neurodegenerative disease involving the depigmentation of substantia nigra and loss of dopaminergic input to basal ganglia. Identify the medication that would worsen rather than improve this patient's condition?

A. Levodopa

B. Bromocriptine

C. Chlorpromazine

D. Pergolide

What is the correct answer?

4

Topic: Alzheimer's Disease
Subject: Medicine
Which one of the following is most predictive of the progression from mild cognitive impairment to frank Alzheimers dementia?

A. Self-reported memory deficits

B. Memory deficits reported by a family member

C. A normal MRI scan of the brain

D. The absence of the apolipoprotein E4 allele

What is the correct answer?

4

Topic: Brain Death
Subject: Medicine
A 57-year-old white male with coronary artery disease suffered a cardiac arrest while jogging 10 days ago. He was resuscitated after 45 minutes but has remained unresponsive and on a ventilator since then.
Which one of the following is required to diagnose brain death in this patient?

A. Hypothermia

B. Continuous mechanical ventilation

C. The absence of spontaneous body movements

D. Electroencephalographic confirmation